LSAT and Law School Admissions Forum

Get expert LSAT preparation and law school admissions advice from PowerScore Test Preparation.

 Administrator
PowerScore Staff
  • PowerScore Staff
  • Posts: 8937
  • Joined: Feb 02, 2011
|
#90602
Complete Question Explanation

Assumption. The correct answer choice is (B).

The argument begins with a conclusion that selling syndicated reruns of a popular show while it is still running on a network can lead to decreased revenue for that network.

The second sentence, which doesn't directly support the conclusion, asserts that the TV show's producers earn a lot from selling syndication rights, but that assertion doesn't do anything to show what the effect of selling such rights will have on the network. So the author continues in the final sentence of the argument by giving evidence from a study that over 80% of programs that are made available as reruns (i.e. in syndication) in the same season as first-run episodes suffer an immediate ratings drop for their first-run episodes.

The problem with using that second premise as support for the conclusion is that it doesn't directly tell us anything about network revenues, which is what the conclusion is all about. The author must assume that there is a connection between a show's ratings and the network's revenues, i.e. that when the ratings drop the network's revenues also drop. This is our prephrase, a classic Supporter Assumption (connecting the premise to something in the conclusion that the premises don't mention directly).

Answer choice (A): Answer choice A does nothing to connect ratings drops to network revenues, moreover it goes well beyond the scope of the argument, because it talks about a specific kind of syndicated show (ones that are sold into syndication early) about which the argument makes no claims at all.

Answer choice (B): This is the correct answer choice. Answer choice B provides the precise connection we were looking for in our prephrase, suggesting that a ratings drop (which the premises discuss) does indeed negatively impact network revenues (which is what the conclusion is about).

Answer choice (C): Answer choice C is well beyond the scope of the argument, because the argument does not depend on, or make claims about, the typical structure of syndication deals. In addition, it provides no foundation for the argument's conclusion that network revenues drop when syndication happens in certain circumstances. Thus, it is incorrect.

Answer choice (D): The argument does not depend on which types of episodes audiences prefer (it rather depends on a connection between ratings and revenue), thus answer choice D is out of scope of the argument and incorrect.

Answer choice (E): The argument only addresses what can happen to network revenues when syndication occurs while a show is still running on its network, thus it does not depend on any broader assumptions about how many programs overall (even ones not still running on their original networks) are sold into syndication. Thus answer choice E is incorrect.
 sofisofi
  • Posts: 23
  • Joined: Mar 31, 2022
|
#97506
Hi
I was wondering what the conclusion of the stimulus would be. I think that's why I had a hard time with this question.
Thanks!
 Robert Carroll
PowerScore Staff
  • PowerScore Staff
  • Posts: 1787
  • Joined: Dec 06, 2013
|
#97537
sofisofi,

The conclusion is the first sentence. The author is claiming that there is a causal connection between selling syndicated reruns of a still-running show and the revenues of the network doing such. The rest of the stimulus purportedly explains why this causal connection exists.

Robert Carroll

Get the most out of your LSAT Prep Plus subscription.

Analyze and track your performance with our Testing and Analytics Package.